Limitando a distribuição de que é o seu padrão normal

8

Seja (Xn) uma sequência de variáveis ​​aleatórias iid N(0,1) . Defina S0=0 e Sn=k=1nXk para n1 . Encontre a distribuição limitadora de

1nk=1n|Sk1|(Xk21)

Esse problema é de um livro de problemas sobre Teoria da Probabilidade, no capítulo sobre o Teorema do Limite Central.

Como Sk1 e Xk são independentes, E(|Sk1|(Xk21))=0 e

V(|Sk1|(Xk21))=E(Sk12(Xk21)2)=E(Sk12)E(Xk21)2)=2(k1)

Observe que os |Sk1|(Xk21) claramente não são independentes. O problema é do Problems in Probability , de Shiryaev , que é baseado no livro do mesmo autor. O livro não parece cobrir o CLT para variáveis ​​correlacionadas. Não sei se há uma sequência estacionária de mixagem escondida em algum lugar ...

Realizei simulações para ter uma ideia da resposta

import numpy as np
import scipy as sc
import scipy.stats as stats
import matplotlib.pyplot as plt

n = 20000 #summation index
m = 2000 #number of samples

X = np.random.normal(size=(m,n))
sums = np.cumsum(X, axis=1)
sums = np.delete(sums, -1, 1)
prods = np.delete(X**2-1, 0, 1)*np.abs(sums)
samples = 1/n*np.sum(prods, axis=1)

plt.hist(samples, bins=100, density=True)
x = np.linspace(-6, 6, 100)
plt.plot(x, stats.norm.pdf(x, 0, 1/np.sqrt(2*np.pi)))
plt.show()

Abaixo está um histograma de 2000 amostras ( n=20.000 ). Parece bastante normalmente distribuído ...

insira a descrição da imagem aqui

Gabriel Romon
fonte
@MartijnWeterings Publiquei isso porque refleti sobre o problema há algum tempo e estou preso. Provavelmente está longe de ser trivial ...
Gabriel Romon
@MartijnWeterings ; portanto,V ( | S k - 1 | ( X 2 k - 1 ) ) = E ( S 2 k - 1 ( X 2 k - 1 ) 2 )E(|Sk1|(Xk21))=0V(|Sk1|(Xk21))=E(Sk12(Xk21)2)
Gabriel Romon
@MartijnWeterings Sim, eu omiti a igualdade trivial por ... x R|x|2=x2xR
Gabriel Romon
O histograma na simulação é uma combinação terrível para a distribuição Normal. Se você não estiver convencido, calcule a curtose.
whuber
@MartijnWeterings Sim, fiz uma omissão embaraçosa no código. Eu o atualizei, assim como o histograma, que parece normal. Você tem uma idéia do valor exato da variação?
Gabriel Romon 20/08/19

Respostas:

1

Quando eu simulo a distribuição, recebo algo que se assemelha a uma distribuição de Laplace. Ainda melhor parece ser um q-Gausiano (os parâmetros exatos que você teria que encontrar usando a teoria).

Acho que seu livro deve conter alguma variação do CLT relacionada a ele (teorema do limite central generalizado q, provavelmente está na Seção 7.6 O teorema do limite central para somas de variáveis ​​dependentes , mas não posso procurá-lo como eu. não tem o livro disponível).

simulação

library(qGaussian)
set.seed(1)
Qstore <- c(0) # vector to store result

n <- 10^6  # columns X_i
m <- 10^2  # rows repetitions

pb <- txtProgressBar(title = "progress bar", min = 0,
                     max = 100, style=3)
for (i in 1:100) {  
  # doing this several times because this matrix method takes a lot of memory
  # with smaller numbers n*m it can be done at once

  X <- matrix(rnorm(n*m,0,1),m)
  S <- t(sapply(1:m, FUN = function(x) cumsum(X[x,])))
  S <- cbind(rep(0,m),S[,-n])
  R <- abs(S)*(X^2-1)
  Q <- t(sapply(1:m, FUN = function(x) cumsum(R[x,])))

  Qstore <- c(Qstore,t(Q[,n]))
  setTxtProgressBar(pb, i)
}
close(pb)

# compute histogram 
x <- seq(floor(min(Qstore/n)), ceiling(max(Qstore/n)), 0.2)
h <- hist(Qstore/(n),breaks = x)

# plot simulation
plot( h$mid, h$density, log = "y", xlim=c(-7,7),
      ylab = "log density" , xlab = expression(over(1,n)*sum(abs(S[k-1])*(X[k]^2-1),k==1,n) ) )

# distributions for comparison
lines(x, dnorm(x,0,1),                   col=1, lty=3)      #normal 
lines(x, dexp(abs(x),sqrt(2))/2,         col=1, lty=2)      #laplace
lines(x, qGaussian::dqgauss(x,sqrt(2),0,1/sqrt(2)), col=1, lty=1)      #qgauss

# further plotting
title("10^4 repetitions with n=10^6")
legend(-7,0.6,c("Gaussian", "Laplace", "Q-Gaussian"),col=1, lty=c(3,2,1),cex=0.8)
Sextus Empiricus
fonte
Em relação ao conteúdo do livro, é melhor que você veja por si mesmo: Volume 1 , Volume 2 . O problema só deve exigir material coberto no Capítulo 3.4
Gabriel Romon
@ GabrielRomon muito obrigado por esses links. Olhando para ele, no meu telefone, não consegui encontrar nada sobre o q-Gaussian ou outras distribuições limitadoras que não são uma distribuição normal. Portanto, ou a distribuição tem uma convergência muito lenta n >> 10 ^ 6 antes de vê- la, ou a pergunta não se encaixa no capítulo (é do livro, eu também não encontrei a pergunta?). Um gráfico dos momentos de ordem superior (em função de n) pode mostrar melhor se a conversão ainda pode ocorrer, mas acho que esse não é um caso típico de CLT.
Sextus Empiricus
1
Este é o problema 3.4.14 no livro de problemas .
Gabriel Romon